【离散数学】 MIT 6.042J 笔记 - Lecture 2 Induction

PDF 文件下载

【离散数学】MIT 6.042J - Fall 2010 - Note for Lecture 2

图片效果

1
2
3

LaTeX \LaTeX LATEX 代码

\documentclass{article}

\usepackage{fancyhdr}
\usepackage{graphicx}
\usepackage{titlesec}
\usepackage{titletoc}
\usepackage{listings}
\usepackage{appendix}
\usepackage{bm}
\usepackage{amsmath}
\usepackage{amsfonts}
\usepackage{amssymb}
\usepackage{amsthm}
\usepackage{multirow}
\usepackage{hyperref}
\usepackage{subfig}
\usepackage{url}
\usepackage{cite}
\usepackage{array}
\usepackage[a4paper, left=2.5cm, right=2.5cm, top=2.65cm, bottom=2.54cm]{geometry}

\newtheorem{definition}{Definition}
\newtheorem{theorem}{Theorem}

\title{\huge \bfseries Mathematics for Computer Science, MIT 6.042J - Fall 2010 - Note for Lecture 2}

\author{\Large Teddy van Jerry}
\date{\today}

\pagestyle{fancy}
\fancyhf{}
\cfoot{\thepage}
\chead{MIT 6.042J Lecture 2 - Induction}

\begin{document}

    \maketitle

    {\Large Lecture 2: Induction}

    \section{Proof by Contradiction}

        \subsection{definition}

            \begin{definition}
                To prove $p$ is True, we assume $p$ is False,
                (i.e. $\lnot p$ is True)
                then use that thypothesis to derive a false hood or contraction.
            \end{definition}

        \subsection{Example 1}
            $$\sqrt{2} \text{ is irrational.}$$

            \begin{proof}
                Assume for the purpose of contradiction
                that $\sqrt{2}$ is rational.
                $$
                \begin{aligned}
                    \Rightarrow & \sqrt{2} = \frac{a}{b}, (a,b)=1 \\
                    \Rightarrow & 2 = \frac{a^2}{b^2} \\
                    \Rightarrow & 2b^2 = a^2 \\
                    \Rightarrow & 2 | a \\
                    \Rightarrow & 2 | b \\
                    \Rightarrow & 2 | (a,b) \\
                    \Rightarrow & (a,b) \neq 1 \\
                    \Rightarrow & \sqrt{2} \text{ is irrational.}
                \end{aligned}
                $$
                
                Irrational number was first discovered by ancient Greek people.
                But they did not like irrational numbers as
                irrational numbers can not be written finitely.
                (in ancient Greek, anything infinite was perceived as bad.)
                
                It caused a stir as it proved inconsistent axioms.
                They covered it up, and denied the result.
            \end{proof}

        \subsection{Example 2}

            $$90 > 92$$

            Proof: by PowerPoint

            \begin{proof}
                We will take two  triangles with total area 90,
                and divide them into four triangles with total area greater than 92.

                By the `conservation of area' axiom,
                this will imply that an area of 90 is larger than an area of 92.
                (Picture omitted)

                So $90 > 92$.
            \end{proof}

            It is false because the rectangle is not drawn correctly.

    \section{Induction axiom}

        \subsection{Definition}
            
            \begin{definition}
                Let $P(n)$ be a predicate.
                If $P(0)$ is not true and $\forall n \in \mathbb{N}$,
                $P(n) \Rightarrow P(n+1)$ is True,
                then $\forall n \in \mathbb{N}$, $P(n)$ is true.
            \end{definition}

            We can view this as a series of dominoes. Each domino knocks over the next.

        \subsection{Example 1}

            $$\forall n \geqslant 0 \text{, } 1+2+3+\cdots + n = \frac{n(n+1)}{2}$$
            or
            $$\forall n \geqslant 0 \text{, } \sum_{i = 1}^{n} = \frac{n(n+1)}{2} $$

            \begin{proof}
            
                Let $P(n)$ be proposition that $\sum_{i = 1}^{n} = \frac{n(n+1)}{2} $.

                Base Case:$$\sum_{i = 1}^{0} = 0 $$

                Inductive Step:
                (For $n \geqslant 0$, show $P(n) \Rightarrow P(n+1)$ is true.)

                Assume $P(n)$ is true for purposes of induction.
                (i.e. assume $1+2+3+\cdots + n = \frac{n(n+1)}{2}$)

                $$1+2+\cdots + n + (n+1) = \frac{n(n+1)}{2} + (n+1) = \frac{(n+1)(n+2)}{2}$$

            \end{proof}

        \subsection{Example 2}

            $$\forall n \in \mathbb{N}, 3\left|(n^3-n)\right.$$

            \begin{proof}
                Let $P(n)$ be $3\left|(n^3-n)\right.$.

                Base Case: $3|(0-0)$.

                Inductive Step: For $n \geqslant 0$, show  $P(n) \Rightarrow P(n+1)$ is true.

                Examine
                $$
                \begin{aligned}
                    (n+1)^3-(n+1) & = n^3+3n^2+3n+1-(n+1) \\
                    & = n^3+3n^2+2n \\
                    & = (n^3-n) + 3(n^2+n) \\
                \end{aligned}
                $$
                
                $$\Rightarrow 3\left|(n+1)^3-(n+1)\right.$$

            \end{proof}

        \subsection{Example 3}

            The induction does not necessarily begin with 0.
            We have:

            Base Case: $P(n)$ is true.

            Inductive Step: $\forall n \geqslant b \text{, } P(n) \Rightarrow P(n+1)$
            
            Conclusion: $\forall n \geqslant b \text{, } P(n)$ is true.

            \subsubsection{A false proof}

                All horses are the same color.

                \begin{proof}
                    Let $P(n)$ be: In any set of $n$ horses,
                    the horses are all the same color.

                    Base Case: One horse is the same color itself.

                    Inductive Step:
                    Assume $P(n)$ is true to prove $P(n+1)$ is true.

                    Consider a set of $n+1$ horses $H_1, H_2, \cdots, H_{n+1}$.

                    Then $H_1, H_2, \cdots, H_{n}$ are the same color.

                    And $H_2, H_3, \cdots, H_{n+1}$ are the same color.

                    Then they share the same color.
                \end{proof}

                The flaw here: $P(1) \Rightarrow P(2)$ is false as from 1 to $n$
                shares no horse with from 2 to $n+1$.
                (Or we can say it is false with the dots. The missing link with $n=1$.)

                Professor: If you don't check the base case,
                you can proof really great stuff.

        \subsection{Example 4}
            $\forall n, \exists$ way to fill a $2^n \times 2^n$ region with
            the L-Shape tiles with a center square missing.

            Way 1:

            \begin{proof}
                Assume $P(n)$ to be $\forall n, \exists$ way to fill a $2^n \times 2^n$ region with
                the L-Shape tiles with a \emph{corner} square missing.

                Base Case has been checked.

                For $n \geqslant 0$, assume $P(n)$ to verify I.H.
                so we need to show $P(n+1)$ is true.

                Then in the case with $n+1$,
                we place the four missing places in the middle,
                fill three of them with a L-Shape tile, leaving a missing square.

            \end{proof}
            
            Way 2: Assume $P(n)$ to be $\forall n, \exists$ way to fill a $2^n \times 2^n$ region with
            the L-Shape tiles with \emph{any} square missing. (Make it stronger.)

    \section*{Appendix}

        ~

        This note is written in \LaTeX.
        
        \LaTeX\ code in my blog: \url{https://blog.csdn.net/weixin_50012998/article/details/113527826}\\

        The webpage of the video: \url{https://www.bilibili.com/video/BV1zt411M7D2?p=2}

\end{document}

% Copyright 2021 Teddy van Jerry

TIP: 文档标题应该是 Induction, LaTeX \LaTeX LATEX 代码中已改正。


ALL RIGHTS RESERVED © 2021 Teddy van Jerry
欢迎转载,转载请注明出处。


See also

Teddy van Jerry 的导航页

  • 0
    点赞
  • 1
    收藏
    觉得还不错? 一键收藏
  • 0
    评论

“相关推荐”对你有帮助么?

  • 非常没帮助
  • 没帮助
  • 一般
  • 有帮助
  • 非常有帮助
提交
评论
添加红包

请填写红包祝福语或标题

红包个数最小为10个

红包金额最低5元

当前余额3.43前往充值 >
需支付:10.00
成就一亿技术人!
领取后你会自动成为博主和红包主的粉丝 规则
hope_wisdom
发出的红包
实付
使用余额支付
点击重新获取
扫码支付
钱包余额 0

抵扣说明:

1.余额是钱包充值的虚拟货币,按照1:1的比例进行支付金额的抵扣。
2.余额无法直接购买下载,可以购买VIP、付费专栏及课程。

余额充值